Annmuriithi
On this page, you find all documents, package deals, and flashcards offered by seller annmuriithi.
- 469
- 0
- 0
Community
- Followers
- Following
3 Reviews received
469 items
Med Surge ExaM 1 Version 2 With Questin And Correct Answers 2024-2025 Intervention torsades de pointes - correct answer-defibrillate (no pulse) Medications for Torsades de Pointes - correct answer-magnesium medications for sinus bradycardia - correct ans
Med Surge ExaM 1 Version 2 
With Questin And Correct 
Answers 
Intervention torsades de pointes - correct answer-defibrillate (no pulse) 
Medications for Torsades de Pointes - correct answer-magnesium 
medications for sinus bradycardia - correct answer-atropine 
epi 
Caused by an occlusion of the RCA - correct answer-inferior wall MI 
inferior wall MI vessel - correct answer-RCA 
caused by an occlusion of the circumflex artery - correct answer-lateral wall MI 
lateral wall MI vessel - correct ...
- Exam (elaborations)
- • 13 pages •
Med Surge ExaM 1 Version 2 
With Questin And Correct 
Answers 
Intervention torsades de pointes - correct answer-defibrillate (no pulse) 
Medications for Torsades de Pointes - correct answer-magnesium 
medications for sinus bradycardia - correct answer-atropine 
epi 
Caused by an occlusion of the RCA - correct answer-inferior wall MI 
inferior wall MI vessel - correct answer-RCA 
caused by an occlusion of the circumflex artery - correct answer-lateral wall MI 
lateral wall MI vessel - correct ...
C985-AnAlytiCAl Methods of heAlth leAder with Question And CorreCt Answers ANOVA - correct answer-Analysis of variance test. Used when there are two or more groups to compare. A group of psychiatric patients are trying three different therapies: counse
C985-AnAlytiCAl 
Methods of heAlth 
leAder with Question 
And CorreCt Answers 
ANOVA - correct answer-Analysis of variance test. Used when there are two or more groups to compare. 
A group of psychiatric patients are trying three different therapies: counseling, medication, and 
biofeedback. You want to see if one therapy is better than others. 
Alternative Hypothesis - correct answer-States that the two groups we are studying are different: this is 
the researcher's hypothesis, the altern...
- Exam (elaborations)
- • 13 pages •
C985-AnAlytiCAl 
Methods of heAlth 
leAder with Question 
And CorreCt Answers 
ANOVA - correct answer-Analysis of variance test. Used when there are two or more groups to compare. 
A group of psychiatric patients are trying three different therapies: counseling, medication, and 
biofeedback. You want to see if one therapy is better than others. 
Alternative Hypothesis - correct answer-States that the two groups we are studying are different: this is 
the researcher's hypothesis, the altern...
RESPIRATORY PRACTICE QUSTION WITH CORRECT ANSWERS PASS A+ While seeing a 62 yo who is hospitalized with CAP, the NP considers that: A. pneumococcal vaccine should be given when antimicrobial therapy has been completed B. pneumococcal vaccine can be giv
RESPIRATORY 
PRACTICE QUSTION 
WITH CORRECT 
ANSWERS PASS A+ 
While seeing a 62 yo who is hospitalized with CAP, the NP considers that: 
A. pneumococcal vaccine should be given when antimicrobial therapy has been completed 
B. pneumococcal vaccine can be given today, and influenza vaccine can be given in 2 weeks 
C. influenza can be given today and antipneumococcal vaccine can be given in 2 weeks 
D. influenza and antipneumococcal vaccine should be given today - correct answer-D 
Risk factors...
- Exam (elaborations)
- • 26 pages •
RESPIRATORY 
PRACTICE QUSTION 
WITH CORRECT 
ANSWERS PASS A+ 
While seeing a 62 yo who is hospitalized with CAP, the NP considers that: 
A. pneumococcal vaccine should be given when antimicrobial therapy has been completed 
B. pneumococcal vaccine can be given today, and influenza vaccine can be given in 2 weeks 
C. influenza can be given today and antipneumococcal vaccine can be given in 2 weeks 
D. influenza and antipneumococcal vaccine should be given today - correct answer-D 
Risk factors...
MPOETC 2024 Exam Question With Correct Answers 100% Upgrand First amendment - Any responsible person capable of taking an oath affirming a criminal complaint correct answer-Freedom of speech, religion, and press and the right to petition is ensured by t
MPOETC 2024 Exam Question 
With Correct Answers 100% 
Upgrand 
First amendment - Any responsible person capable of taking an oath affirming a criminal complaint 
correct answer-Freedom of speech, religion, and press and the right to petition is ensured by the 
Second amendment - correct answer-Right to bear arms 
Third amendment - correct answer-The government may not house soldiers in private homes without 
consent of the owner 
Fourth Amendment - correct answer-Search and seizure 
Fifth ame...
- Exam (elaborations)
- • 12 pages •
MPOETC 2024 Exam Question 
With Correct Answers 100% 
Upgrand 
First amendment - Any responsible person capable of taking an oath affirming a criminal complaint 
correct answer-Freedom of speech, religion, and press and the right to petition is ensured by the 
Second amendment - correct answer-Right to bear arms 
Third amendment - correct answer-The government may not house soldiers in private homes without 
consent of the owner 
Fourth Amendment - correct answer-Search and seizure 
Fifth ame...
OECP Exam Question with Correct Answer 100% Complete Solutions A crane operator must be certified for the _____ and ______ of the crane they will operate - correct answer-Type and capacity What is the horizontal distance from the center of rotation t
OECP Exam 
Question with 
Correct Answer 
100% Complete 
Solutions 
A crane operator must be certified for the _____ and ______ of the crane they will operate - correct 
answer-Type and capacity 
What is the horizontal distance from the center of rotation to the center of the suspended load called? - 
correct answer-Load radius 
A jib is primarily used for ? - correct answer-Height 
What is the principle of leverage? - correct answer-Weight x distance = distance x weight 
Whats the formula ...
- Exam (elaborations)
- • 22 pages •
OECP Exam 
Question with 
Correct Answer 
100% Complete 
Solutions 
A crane operator must be certified for the _____ and ______ of the crane they will operate - correct 
answer-Type and capacity 
What is the horizontal distance from the center of rotation to the center of the suspended load called? - 
correct answer-Load radius 
A jib is primarily used for ? - correct answer-Height 
What is the principle of leverage? - correct answer-Weight x distance = distance x weight 
Whats the formula ...
OECP Exam question with correct answers 100% Complete Solutions What are the four different types of crane signals? - correct answer-Hand Voice Audible New The shaded area on a load chart indicates _______? - correct answer-A structural chart The mini
OECP Exam 
question with 
correct answers 
100% Complete 
Solutions 
What are the four different types of crane signals? - correct answer-Hand 
Voice 
Audible 
New 
The shaded area on a load chart indicates _______? - correct answer-A structural chart 
The minimum distance to a power line of unknown voltage is ? - correct answer-50' 
When hoisting personnel and have known voltage, can you use Table A and get closer than 20'? - correct 
answer-Never! 
What is the minimum tale swing clearan...
- Exam (elaborations)
- • 22 pages •
OECP Exam 
question with 
correct answers 
100% Complete 
Solutions 
What are the four different types of crane signals? - correct answer-Hand 
Voice 
Audible 
New 
The shaded area on a load chart indicates _______? - correct answer-A structural chart 
The minimum distance to a power line of unknown voltage is ? - correct answer-50' 
When hoisting personnel and have known voltage, can you use Table A and get closer than 20'? - correct 
answer-Never! 
What is the minimum tale swing clearan...
AANP ExAm QuEstioN with CorrECt ANswErs (2024-2025) FNP PAss A+ A 32-year-old woman presents to the clinic with a persistent, worsening cough for 2 weeks. She reports coughing spells and states that it is difficult to stop coughing once it starts. The
AANP ExAm 
QuEstioN with 
CorrECt ANswErs 
() FNP 
PAss A+ 
A 32-year-old woman presents to the clinic with a persistent, worsening cough for 2 weeks. She reports 
coughing spells and states that it is difficult to stop coughing once it starts. The nurse practitioner 
suspects acute bronchitis due to pertussis because of a local outbreak. Which of the following is the 
most appropriate plan of care? 
A. 
Educate the patient about symptomatic relief 
B. 
Follow up in 1 week if the cough per...
- Exam (elaborations)
- • 70 pages •
AANP ExAm 
QuEstioN with 
CorrECt ANswErs 
() FNP 
PAss A+ 
A 32-year-old woman presents to the clinic with a persistent, worsening cough for 2 weeks. She reports 
coughing spells and states that it is difficult to stop coughing once it starts. The nurse practitioner 
suspects acute bronchitis due to pertussis because of a local outbreak. Which of the following is the 
most appropriate plan of care? 
A. 
Educate the patient about symptomatic relief 
B. 
Follow up in 1 week if the cough per...
NUR 170 Exam 2 with QUEstioN aNd CoRRECt aNswERs Pass a+ What are some triggers for clusters - correct answer-Usually lifestyle related -Stress, anger, anxiety Can be different for each patient What does an abortive treatment mean? - correct answer-Stop
NUR 170 Exam 2 with QUEstioN 
aNd CoRRECt aNswERs Pass a+ 
What are some triggers for clusters - correct answer-Usually lifestyle related 
-Stress, anger, anxiety 
Can be different for each patient 
What does an abortive treatment mean? - correct answer-Stop it before it starts (usually stop at aura) 
What is the problem with Triptan medications? - correct answer-They can cause ischemia in the heart 
which can cause a heart attack 
What are some medications that can be prescribed for patients ...
- Exam (elaborations)
- • 17 pages •
NUR 170 Exam 2 with QUEstioN 
aNd CoRRECt aNswERs Pass a+ 
What are some triggers for clusters - correct answer-Usually lifestyle related 
-Stress, anger, anxiety 
Can be different for each patient 
What does an abortive treatment mean? - correct answer-Stop it before it starts (usually stop at aura) 
What is the problem with Triptan medications? - correct answer-They can cause ischemia in the heart 
which can cause a heart attack 
What are some medications that can be prescribed for patients ...
UWM Bio ExaM 4 2024- 2025 QUEstion and CorrECt ansWErs Upgrand What are Gnathostomata? - correct answer-The Jawed Vertebrates -Cartilaginous fishes, bony fishes, amphibians, reptiles, birds, and mammals -Also have paired fins, efficient kidneys, and sop
UWM Bio ExaM 4 2024- 
2025 QUEstion and 
CorrECt ansWErs 
Upgrand 
What are Gnathostomata? - correct answer-The Jawed Vertebrates 
-Cartilaginous fishes, bony fishes, amphibians, reptiles, birds, and mammals 
-Also have paired fins, efficient kidneys, and sophisticated buoyancy control mechanisms 
-60,000 species in 7 taxonomic classes 
What are Class Chondrichthyes? - correct answer-The Cartilaginous Fishes 
-1,110 species of mostly marine predators and scavengers 
-Internal skeleton of carti...
- Exam (elaborations)
- • 16 pages •
UWM Bio ExaM 4 2024- 
2025 QUEstion and 
CorrECt ansWErs 
Upgrand 
What are Gnathostomata? - correct answer-The Jawed Vertebrates 
-Cartilaginous fishes, bony fishes, amphibians, reptiles, birds, and mammals 
-Also have paired fins, efficient kidneys, and sophisticated buoyancy control mechanisms 
-60,000 species in 7 taxonomic classes 
What are Class Chondrichthyes? - correct answer-The Cartilaginous Fishes 
-1,110 species of mostly marine predators and scavengers 
-Internal skeleton of carti...
ATI PhArmAcology ProcTored,PhArmAcology ATI ProcTored exAm wITh QuesTIon And correcT Answers 2024-2025 E. When the same pathway metabolizes two medications, they compete for metabolism, thereby increasing the concentration of one or both medications.
ATI PhArmAcology 
ProcTored,PhArmAcology ATI 
ProcTored exAm wITh QuesTIon And 
correcT Answers 
E. When the same pathway metabolizes two medications, they compete for metabolism, thereby 
increasing the concentration of one or both medications. This requires decreasing the dosage of one or 
both 
A nurse is preparing to administer eye drops to a client. Which of the following actions should the nurse 
take? (Select all that apply.) 
A. Have the client lie on her side. 
B. Ask the client ...
- Exam (elaborations)
- • 135 pages •
ATI PhArmAcology 
ProcTored,PhArmAcology ATI 
ProcTored exAm wITh QuesTIon And 
correcT Answers 
E. When the same pathway metabolizes two medications, they compete for metabolism, thereby 
increasing the concentration of one or both medications. This requires decreasing the dosage of one or 
both 
A nurse is preparing to administer eye drops to a client. Which of the following actions should the nurse 
take? (Select all that apply.) 
A. Have the client lie on her side. 
B. Ask the client ...
(COMPLETE) ATI RN LEADERSHIP PROCTORED EXAM 2019 VERSION 1, 2,3,4,5 & 6 (420 QUESTIONS ALL WITH CORRECT ANSWERS)| GUARANTEE A+ SCORE |VERIFIED|TEST BANK FINAL EXAM 2023 LATEST UPDATE VERSION 1 1. A nurse is assigned the following four clients for the cu
NRSG=113 Test-Bank = Maternal Child Nursing Care=by Perry -6th Edition- NEW UPDATE 2023/2024 NRSG 113 Test Bank - Maternal Child Nursing Care by Perry (6th Edition) NEW UPDATE 2022/2023 Table of Contents Table of Contents 1 Chapter 01: 21st Century Mate
Introductory Mental Health Nursing 4th Edition TEST BANK WITH THE CORRECT ANSWER KEY by Donna Womble Introductory Mental Health Nursing 4th Edition Womble Kincheloe Test Bank Chapter 1 Mental Health and Mental Illness 1. When studying mental health and